Inscription / Connexion Nouveau Sujet
Niveau autre
Partager :

Les séries

Posté par
lamhaya
03-01-10 à 01:08

Bonsoir, j'ai vraiment besoin d'aide pour cet exercice, voici l'enoncé :

On considère les suites (u_n) et (v_n) (tel que n1)

(u_n)= 1 + \frac{1}{2} + \frac{1}{3} + ... + \frac{1}{n} - ln(n)

et (v_n)= (u_n)- \frac{1}{n}

1/ Pour tout entier non nul, montrer que \frac{1}{n+1}<=\int_n^{(n+1)}\frac{dx}{x}<=\frac{1}{n}

2/ Montrer que (u_n) et (v_n)( tel que n1) sont monotones.

Merci pour votre aide

Posté par
gui_tou
re : Les séries 03-01-10 à 10:21

Salut

ou bloques-tu ?

1) simplement étude de la fonction f(x)=1/x (décroissance etc)

2) calcule un+1-un tout simplement



Vous devez être membre accéder à ce service...

Pas encore inscrit ?

1 compte par personne, multi-compte interdit !

Ou identifiez-vous :


Rester sur la page

Inscription gratuite

Fiches en rapport

parmi 1674 fiches de maths

Désolé, votre version d'Internet Explorer est plus que périmée ! Merci de le mettre à jour ou de télécharger Firefox ou Google Chrome pour utiliser le site. Votre ordinateur vous remerciera !